Đến nội dung

Kamii0909 nội dung

Có 155 mục bởi Kamii0909 (Tìm giới hạn từ 28-04-2020)



Sắp theo                Sắp xếp  

#670566 $ \frac{a^2b+b^2c+c^2a}{a^3+b^3+c^3} \leq \frac{ab+bc+ca}...

Đã gửi bởi Kamii0909 on 06-02-2017 - 20:14 trong Bất đẳng thức - Cực trị

Cho các số thực không âm $a,b,c$. Chứng minh rằng
$$ \frac{a^2b+b^2c+c^2a}{a^3+b^3+c^3} \leq \frac{ab+bc+ca}{a^2+b^2+c^2} + \frac{3 \sum (a-b)^2}{4 (a+b+c)^2}$$



#672103 $ \sum \dfrac{a^2}{b+c}+6(ab+bc+ca) \geq \dfrac{5}{2...

Đã gửi bởi Kamii0909 on 19-02-2017 - 17:39 trong Bất đẳng thức - Cực trị

Chứng minh bất đẳng thức sau với $a,b,c \geq 0,a+b+c=1, k=\dfrac{8}{27} ( 5 \sqrt{10}-13)$
$$ \sum \dfrac{a^2}{b+c}+6(ab+bc+ca) \geq \dfrac{5}{2} +k \dfrac{\sum (a^2b-ab^2)^2}{(a^2+b^2+c^2)^2}$$



#669788 $ \sum \sqrt{a+b+\sqrt{ca}+\sqrt{cb}} \geq k(...

Đã gửi bởi Kamii0909 on 24-01-2017 - 22:29 trong Bất đẳng thức - Cực trị

Cho các số thực không âm $a,b,c$ thỏa mãn $a^2+b^2+c^2=2(ab+bc+ca)$
Tìm hằng số k tốt nhất sau cho bất đẳng thức sau luôn đúng $$ \sum \sqrt{a+b+\sqrt{ca}+\sqrt{cb}} \geq k(\sum \sqrt{a})$$



#675570 $$\prod \left( \dfrac{a}{b}+2...

Đã gửi bởi Kamii0909 on 28-03-2017 - 22:52 trong Bất đẳng thức - Cực trị

Cho các số thực dương $a,b,c$. Cmr
$$ ( \dfrac{a}{b}+2)( \dfrac{b}{c} +2)( \dfrac{c}{a}+2 ) + \dfrac{117(ab+bc+ca)}{4(a^2+b^2+c^2)} \geq \frac{107}{2}$$



#664361 $\boldsymbol{Topic}$ Các bài toán số học HSG Toán 8 + 9

Đã gửi bởi Kamii0909 on 11-12-2016 - 13:14 trong Số học


ĐỀ BÀI

$\boxed{1}$: (Hellenic Mathematical Competitions 2013)
Xác định tất cả các bộ ba số nguyên dương (x, y, z) thỏa mãn phương trình sau đây:


$\frac{1}{x}+\frac{2}{y}-\frac{4}{z}=1$

$\boxed{2}$: (Hellenic Mathematical Competitions 2013) Xác định tất cả các số nguyên x và y thỏa mãn phương trình sau đây:


$y=2x^2+5xy+3y^2$

$\boxed{3}$ Tìm nghiệm nguyên của phương trình


$x^3+y^3=(x+y)^2$

Bài 1.
Biến đổi tương đương ta có
$x=\frac{yz}{yz+4y-2z}$
Nên $yz+4y-2z|yz$.
Từ đó có $z \geq 2y$
Nếu $z=2y$ ta có bộ $(1,t,2t)$ thỏa mãn.
Xét $z>2y$
Lại có $yz+4y-2z|2z-4y$ nên $(y-4)(z+8) \leq 32$(*)
Nếu $y \geq 6$ thì $z \geq 12$.
Khi đó dễ thấy (*) vô lý.
Vậy $y \leq 5$
Đến đây dễ rồi.

Bài 2.
Xét $\Delta$ theo $x$ ta có
$\Delta = y^2+8y=(y+4)^2-16=a^2$ với $a$ là số tự nhiên
$\Leftrightarrow (y+4-a)(y+4+a)=16$

Bài 3.
Dễ thấy có nghiệm $(x,y)=(t,-t)$
Xét TH $x+y$ khác $0$.
Biến đổi pt về dạng $x^2-xy+y^2=x+y$
$\Leftrightarrow x^2-x(y+1)+y^2-y=0$
Coi đây là phương trình bậc 2 ẩn $x$ có $\Delta = -3y^2+6y+1 \geq 0$
$\Leftrightarrow 3y^2-6y-1 \leq 0$
Bất phương trình có nghiệm nguyên $y=0,1,2$.
Thế vào ta có $x$.



#658528 $\boldsymbol{Topic}$ Các bài toán số học HSG Toán 8 + 9

Đã gửi bởi Kamii0909 on 20-10-2016 - 15:18 trong Số học

Bài 1 :  Nhận xét nếu $a \ge 3$ lúc đó $a+b+c>a+b>3$ 
Khi đó $VT<1$ (vô lí) . Nếu $a=1$ cũng dẫn đến vô lí vì $\frac{1}{a+b}+\frac{1}{a+b+c}=0$ 
Do đó $a=2$ . Biến đổi phương trình về thành $b^2+4b+4+(b+2)c=2c+4b+8 \Leftrightarrow bc+b^2+8b+12=0$ vô lí vì $a,b,c nguyên dương$ 
Bài 2 : Xét số dư của $a,b,c$ cho $3$ ta có đpcm |
 

Câu 1 bạn biến đổi nhầm khúc cuối kìa  :icon6:  :icon6:

Mình làm như sau 
Dễ có $\frac{1}{a}> \frac{1}{a+b}> \frac{1}{a+b+c}\Rightarrow \frac{1}{a}> \frac{1}{3}\Rightarrow a< 3$

Mà $a=1$ cũng vô lý vậy $a=2$

Nhân lên ta có $b(b+c)=4$ mà $b< b+c\Rightarrow b=1,c=3$

Câu 2 $\Leftrightarrow (a+bc)(b+ac)=101^{n}$




#667003 $\frac{1}{a^{2}}+\frac{1...

Đã gửi bởi Kamii0909 on 04-01-2017 - 23:21 trong Bất đẳng thức và cực trị

Đặt $(a,b,c) \rightarrow \left( \dfrac{1}{x},\dfrac{1}{y},\dfrac{1}{z} \right)$
Ta biến đổi bđt cần cm về
$$x^2+y^2+z^2+3 \geq 2(\frac{1}{x}+\frac{1}{y}+\frac{1}{z})\Leftrightarrow x^2+y^2+z^2+2xyz+1 \geq 2(xy+yz+xz)$$
Bất đẳng thức cuối quen thuộc.



#666998 $\frac{a^{2}}{c}+\frac{b^...

Đã gửi bởi Kamii0909 on 04-01-2017 - 23:15 trong Bất đẳng thức và cực trị

làm gì có a + b + c = 1 hả bạn !!

Có đấy bạn. Nếu không có chọn $a=b=c$ có ngay điều vô lý.



#666905 $\frac{a^{2}}{c}+\frac{b^...

Đã gửi bởi Kamii0909 on 04-01-2017 - 12:34 trong Bất đẳng thức và cực trị

Ta có $$\sum \frac{a^2}{c}= \sum \frac{a^4}{a^2c} \geq \frac{(\sum a^2)^2}{\sum a^2c}$$
Ta phải chứng minh
$$\sum a^2 \geq 3(\sum a^2c) \Leftrightarrow (\sum a^2)(\sum a) \geq 3(\sum a^2c) \Leftrightarrow \sum a(a-b)^2 \geq 0$$



#670334 $\frac{a}{a^2+1}+\frac{b}{b^2+1}+\frac{c}{c^2+1}\leq...

Đã gửi bởi Kamii0909 on 29-01-2017 - 13:19 trong Bất đẳng thức và cực trị

CM tương đương.
chuyển vế: $(\frac{a}{a^2+1}-\frac{3}{10})+(\frac{b}{b^2+1}-\frac{3}{10})+(\frac{c}{c^2+1}-\frac{3}{10})\leq 0<=>\sum( \frac{-3a^2+10a-3}{a^2+1})$(luôn đúng).
Dấu "=" xảy ra khi x=y=3.

Hãy đánh giá thử bất đẳng thức cuối xem vì sao nó đúng?

$$\sum \dfrac{a}{a^2+1} \leq \frac{9}{10} $$
$$\Leftrightarrow \sum \dfrac{a}{a^2+1} \leq \frac{9}{10} + \sum \dfrac{6(3a-1)}{25}$$
$$\Leftrightarrow \sum \dfrac{(3a-1)^2(4a+3)}{50(a^2+1)}\geq 0$$
Hiển nhiên đúng.



#676639 $\prod (a^{2}+b^{2})\leq \frac{1...

Đã gửi bởi Kamii0909 on 08-04-2017 - 18:51 trong Bất đẳng thức - Cực trị

Nhưng như vầy thì em dồn $c$ về $0$ !

Em chưa hiểu ý anh lắm. Em đang dồn $b \rightarrow 0$. Nếu dồn $c \rightarrow 0$ thì có lẽ là $f(a+c,b,0)$ chính xác hơn.



#676603 $\prod (a^{2}+b^{2})\leq \frac{1...

Đã gửi bởi Kamii0909 on 08-04-2017 - 00:01 trong Bất đẳng thức - Cực trị

Biểu thức $f(a,b,c)$ của em là gì ? Nếu dồn biến theo kiểu này thì sẽ chọn $c$ là số nhỏ nhất, anh thử nhẩm với $f(a,b,c) = (a^2+b^2)(b^2+c^2)(c^2+a^2)$ khi xét $f(a,b,c) \leqslant f(a+b,c,0)$ thì hai đại lượng trội nhất là $a^3b,ab^3$ nằm bên trái dấu $\leqslant $ nên có thể bất đẳng thức này sai.

Em chọn $c$ là số lớn nhất.
Ta nhân 2 đánh giá sau
$c^2[(a+b)^2+c^2]=c^4+c^2a^2+c^2b^2+2c^2ab \geq (c^2+a^2)(c^2+b^2)$
Và $(a+b)^2 \geq (a^2+b^2)$



#676727 $\prod (a^{2}+b^{2})\leq \frac{1...

Đã gửi bởi Kamii0909 on 09-04-2017 - 13:15 trong Bất đẳng thức - Cực trị

Em định nghĩa $f(a,b,c) = (a^2+b^2)(b^2+c^2)(c^2+a^2)$ thì $f(a+b,c,0)$ tức là thay $a = a + b,\, b = c$ và $c = 0.$ Tức $c$ là nhỏ nhất.

Thực ra nó chỉ là mặt quy ước, ta có thể thấy $f(a+c,b,0)$ và $f(a+b,c,0)$ hay $f(0,b,a+c)$ là tương đương hết sau phép đặt ẩn $(a+c,b) \rightarrow (x,y)$ hay $(a+b,c) \rightarrow (x,y)$
Ta chọn vị trí các số sao cho chứng minh dễ nhất có thể mà thôi. Nó không ảnh hưởng dù $c$ max hay min. Bài toán này có thể chọn thứ tự thoải mái do tính đối xứng.



#675568 $\prod (a^{2}+b^{2})\leq \frac{1...

Đã gửi bởi Kamii0909 on 28-03-2017 - 22:46 trong Bất đẳng thức - Cực trị

Cmr $$f(a,b,c) \leq f(a+b,c,0)$$.



#666871 $\sum \dfrac{abc+b+c-a}{a^2+1} \geq...

Đã gửi bởi Kamii0909 on 03-01-2017 - 22:23 trong Bất đẳng thức và cực trị

Với $a,b,c \geq 0$ chứng minh rằng
$\dfrac{abc+b+c-a}{a^2+1}+\dfrac{abc+c+a-b}{b^2+1} +\dfrac{abc+a+b-c}{c^2+1} \geq a+b+c$



#673315 $\sum \frac{a(b+c)}{b^2+bc+c^2}$

Đã gửi bởi Kamii0909 on 03-03-2017 - 01:52 trong Bất đẳng thức - Cực trị

1/CMR:

    a,b,c>0 

          $\sum \frac{a(b+c)}{b^2+bc+c^2}$$\geq 2$

Theo AM-GM $$4(a^2+ab+b^2)(ab+bc+ca) \leq (a+b)^2(a+b+c)^2$$
Bất đẳng thức cần chứng minh đưa về $$\frac{a}{b+c} +\frac{b}{a+c} +\frac{c}{a+b} \geq \frac{(a+b+c)^2}{2(ab+bc+ca)}$$

Hiển nhiên theo C-S. 

Spoiler




#670624 $\sum \frac{1}{\sqrt{a+2b+6}}\leq 1$

Đã gửi bởi Kamii0909 on 07-02-2017 - 13:52 trong Bất đẳng thức - Cực trị

Cho các số thực dương $a, b, c$ thỏa mãn $abc=1.$ Chứng minh rằng $\frac{1}{\sqrt{a+2b+6}}+\frac{1}{\sqrt{b+2c+6}}+\frac{1}{\sqrt{c+2a+6}}\leq 1.$


$$\sum_{cyc} \frac{1}{\sqrt{a+2b+6}} \leq \sqrt{ 3 \sum_{cyc} \frac{1}{a+2b+6}} \leq \sqrt{ 3 \left( \dfrac{4}{9} \sum_{cyc} \dfrac{1}{a+2b+3} + \frac{1}{9} \right)} \leq \sqrt{3 \left( \dfrac{4}{9} \sum_{cyc} \dfrac{1}{2( \sqrt{ab}+ \sqrt{b}+1)} + \frac{1}{9} \right)} = 1$$



#657034 $\sum \frac{4}{a+b} \leq \frac{1}{a}+\frac{1}{b...

Đã gửi bởi Kamii0909 on 07-10-2016 - 20:40 trong Bất đẳng thức và cực trị

Bài 6 $2ab+6bc+2ac=7abc$ <=> $\frac{2}{c}+\frac{6}{a}+\frac{2}{b}=7\Leftrightarrow 6x+2y+2z=7$

C=$\frac{4}{y+2x}+\frac{9}{z+4x}+\frac{4}{y+z}\geq \frac{\left (2+3+2 \right )^{2}}{6x+2y+2z}=7$ 




#671492 $\sum (a^3-b^3)^2 \geq 3abc(a-b)(b-c)(c-a)$

Đã gửi bởi Kamii0909 on 13-02-2017 - 18:30 trong Bất đẳng thức - Cực trị

Bất đẳng thức vẫn đúng mà không cần điều kiện $a^5b+b^5c+c^5a=a^4b^2+b^4c^2+c^4a^2.$

Anh cho em tham khảo lời giải tổng quát? Và đặc biệt là hằng số tốt nhất. E có thử tìm nhưng nó khá là khó và chỉ dừng ở 6.
Em có thử với $a^5b+b^5c+c^5a \geq a^4b^2+b^4c^2+c^4a^2$ và cũng đã chứng minh được bất đẳng thức đúng tuy nhiên chưa làm được phần còn lại.



#671421 $\sum (a^3-b^3)^2 \geq 3abc(a-b)(b-c)(c-a)$

Đã gửi bởi Kamii0909 on 13-02-2017 - 02:50 trong Bất đẳng thức - Cực trị

Cho $a,b,c$ là các số thực thỏa mãn $a^5b+b^5c+c^5a=a^4b^2+b^4c^2+c^4a^2$
Chứng minh rằng
$(a^3-b^3)^2+(b^3-c^3)^2+(c^3-a^3)^2 \geq 6abc(a-b)(b-c)(c-a)$



#658044 $\sum\sqrt[3]{\frac{c}{b+a}}>\frac{\sqrt[3]...

Đã gửi bởi Kamii0909 on 16-10-2016 - 11:33 trong Bất đẳng thức và cực trị

1/ Cho a,b,c>0 thỏa mãn: $\frac{1}{1+a}+\frac{1}{1+b}+\frac{1}{1+c}\geq 2$CMR: $abc\leq \frac{1}{8}$
2/ Cho a,b >0, thỏa mãn a+b=1. CMR: $(a+\frac{1}{a})^2+(b+\frac{1}{b})^2\geq \frac{25}{2}$
3/ Cho a,b,c >0 thỏa mãn: a+b+c+d=1. CMR:
$(a+\frac{1}{a})^2+(b+\frac{1}{b})^2+(c+\frac{1}{c})^2+(d+\frac{1}{d})^2\geq \frac{289}{4}$
4/ Cho a,b,c >0. CMR: $\sqrt[3]{\frac{a}{b+c}}+\sqrt[3]{\frac{b}{a+c}}+\sqrt[3]{\frac{c}{b+a}}>\frac{\sqrt[3]{2}}{2}$

Ta có bất đẳng thức sau với mọi a,b,c không âm

$\sum \sqrt[3]{\frac{a}{b+c}}\geq \sum \sqrt{\frac{a}{b+c}}\geq 2$

Dấu bằng xảy ra khi 1 biến =0 và 2 biến còn lại bằng nhau




#673314 $(a+b)^2(b+c)^2(a+c)^2\geq abc(a+2b+c)(a+2c+b)(2a+b+c)$

Đã gửi bởi Kamii0909 on 03-03-2017 - 01:10 trong Bất đẳng thức - Cực trị

Xét $3(a+b)^2(b+c)^2(c+a)^2-3abc(a+b+2c)(b+c+2a)(c+a+2b)=q^2(p^2-3q)+(3q+2p^2)(q^2-3pr) \geq 0$




#691650 $(x^{2}+y+f(y))=f(x)^{2}+a, \forall x,y \i...

Đã gửi bởi Kamii0909 on 27-08-2017 - 02:22 trong Phương trình hàm

Chuẩn phải là $ P(x,y): f(x^2+y+f(y))=f^2(x)+ay, \forall x,y \in \mathBB{R}$
Nếu $a=0$ dễ chỉ ra 2 hàm $f(x) =0,f(x)=1$ thoả mãn. 
Xét $a \neq 0$ thì ta có $f$ toàn ánh. 

Nếu $f(a)=0$ ta chỉ ra $a=0$
Thật vậy, ta có
$P(b,y)-P(-b,y): f(b)=f(-b)=0$
$P(0,b)+P(0,-b): f(b)+f(-b)=2f^2(0)=0$
Tức là $f(0)=0$, từ đó $P(0,b): ab=0 \Leftrightarrow b=0$

 

Chứng minh $f(x^2)=bx^2, b \geq 0, b^2+b=a$
$P(x,0):f(x^2)=f^2(x)$
Từ đây $f$ đơn ánh trên từng khoảng $(0,+ \infty)$ và $(- \infty,0)$ và $f(x) \geq 0, \forall x \geq 0$
Từ đó $f(x^2)+x^2 \geq 0,\forall x$
$P(0,x):f(x+f(x))=ax$
$P( \sqrt{x^2+f(x^2)},y^2): f(x^2+y^2+f(x^2)+f(y^2))=a(x^2+y^2)$
Và $P(0,x^2+y^2): f(x^2+y^2+f(x^2+y^2))=a(x^2+y^2)$
Do $f$ đơn ánh trên $(0, +\infty)$ nên $f(x+y)=f(x)+f(y),\forall x,y \geq 0$
Kết hợp với $f(x) \geq 0,\forall x \geq 0$ 
Và xét $f$ trên $(0, +\infty)$ dễ thu được $f(x)=bx,\forall x \geq 0(b > 0)$
Trong đó $b^2+b=a$

Bây giờ ta chứng minh $f(-x^2) \leq 0, \forall x$ 
Giả sử $\exists x,f(-x^2) \geq 0$ 
$P(x,-x^2):f(f(-x^2))=f(x^2)-ax^2=-b^2x^2 <0$
Mà $f(-x^2)>0$ nên $f(f(-x^2)) >0$
Mâu thuẫn, tức là $f(-x^2) \leq 0$

 

$f(x)=-f(-x)$

Ta có $f(x^2)=f(-x)^2=f(x)^2$

Dễ thấy 2 số $x$ và $-x$ khác dấu $ \forall x \neq 0$ nên $f(x)$ và $f(-x)$ cũng khác dấu.

Nói cách khác $f(x) \neq f(-x), \forall x \neq 0$ dẫn tới $f(x)=-f(-x), \forall x$

$f(x^2)=bx^2$ nên $f(x)=bx,\forall x$

Trong đó $b^2+b=a \geq 0$

Với $a>0$ ta tìm được nghiệm dương duy nhất của $b$ là $\dfrac{2a}{1+ \sqrt{1+4a}}$

Kết luận $a>0$




#658830 $(x^2+4y^2+28)^2=17(x^4+y^4+14y^2+49)$

Đã gửi bởi Kamii0909 on 22-10-2016 - 21:38 trong Số học

Ừ đúng rồi :) mình khai triển hết ra rồi chuyển thành nhân tử đó

Có cách nào đẹp hơn mà chỉ xét module không nhỉ  :(  :(

Cách này nặng chất biến đổi quá




#691649 $(x_n):\left\{\begin{matrix} ...\...

Đã gửi bởi Kamii0909 on 27-08-2017 - 00:36 trong Dãy số - Giới hạn

Chứng minh theo quy nạp rằng $x_n \leq \dfrac{25}{4}- \dfrac{5}{n}, n \geq 1$
Kiểm tra với $n=1,2$ đúng. 
Sử dụng công thức truy hồi, ta cần chỉ ra 
$$x_{n+1}<x_{n}+\dfrac{5}{n^3} \leq \dfrac{25}{4} - \dfrac{5}{n} + \dfrac{5}{n^3} \leq \dfrac{25}{4}- \dfrac{5}{n+1}, \forall n \geq 2$$

$$\Leftrightarrow n^2 \geq n+1,\forall n \geq 2$$

Từ đó ta có đpcm.